- Fri Jan 20, 2017 12:00 am
#47095
Complete Question Explanation
(The complete setup for this game can be found here: lsat/viewtopic.php?t=11700)
The correct answer choice is (E)
Global Could Be True questions, such as this one, are often best left until the end of the game to complete because answering these questions is often much easier after you have seen several solutions to the game (for example, question #4 in this game contains three answers that can be used to eliminate answers in this question). Thus, unless the game is an Identify the Templates or Possibilities game, skip these questions and return to them at the end of the game. If you do attack this question in its natural position, make sure to use the hypotheticals produced by the first two questions. For example, the solution to question #1 eliminates answer choice (A), and the solution to question #2 eliminates answer choice (B). By using that approach, you will have reduced the amount of work you have to do to determine the correct answer.
Answer choice (A): As mentioned above, the solution to question #1 (answer choice (D), shows that T can be scheduled earlier than L. The work in question #2 also proves the same point.
Answer choice (B): As mentioned above, the work in question #2 shows that L can be scheduled earlier than P.
Answer choice (C): This answer choice can be disproved by the following hypothetical:
As shown above, G can be scheduled earlier than T.
Answer choice (D): This answer choice can be disproved by the following hypothetical:
As shown above, W can be scheduled earlier than T.
Answer choice (E): This is the correct answer choice. G must be scheduled for one of the last three days, and, as discussed in the setup, W must be scheduled for one of the first three days. Because W must always be scheduled earlier than G, this is the correct answer choice. If the logic behind this is unclear, try a hypothetical: there is not enough room for G to be scheduled ahead of W because W must also be scheduled ahead of L, and S must be scheduled in the last three days as well.
(The complete setup for this game can be found here: lsat/viewtopic.php?t=11700)
The correct answer choice is (E)
Global Could Be True questions, such as this one, are often best left until the end of the game to complete because answering these questions is often much easier after you have seen several solutions to the game (for example, question #4 in this game contains three answers that can be used to eliminate answers in this question). Thus, unless the game is an Identify the Templates or Possibilities game, skip these questions and return to them at the end of the game. If you do attack this question in its natural position, make sure to use the hypotheticals produced by the first two questions. For example, the solution to question #1 eliminates answer choice (A), and the solution to question #2 eliminates answer choice (B). By using that approach, you will have reduced the amount of work you have to do to determine the correct answer.
Answer choice (A): As mentioned above, the solution to question #1 (answer choice (D), shows that T can be scheduled earlier than L. The work in question #2 also proves the same point.
Answer choice (B): As mentioned above, the work in question #2 shows that L can be scheduled earlier than P.
Answer choice (C): This answer choice can be disproved by the following hypothetical:
As shown above, G can be scheduled earlier than T.
Answer choice (D): This answer choice can be disproved by the following hypothetical:
As shown above, W can be scheduled earlier than T.
Answer choice (E): This is the correct answer choice. G must be scheduled for one of the last three days, and, as discussed in the setup, W must be scheduled for one of the first three days. Because W must always be scheduled earlier than G, this is the correct answer choice. If the logic behind this is unclear, try a hypothetical: there is not enough room for G to be scheduled ahead of W because W must also be scheduled ahead of L, and S must be scheduled in the last three days as well.
You do not have the required permissions to view the files attached to this post.